Gönderen Konu: Kesirli Kısmın İntegrali  (Okunma sayısı 1978 defa)

Çevrimdışı Metin Can Aydemir

  • G.O Genel Moderator
  • Geo-Maniac
  • ********
  • İleti: 1139
  • Karma: +9/-0
Kesirli Kısmın İntegrali
« : Ağustos 20, 2019, 06:17:21 ös »
$\{x\}$, $x$'in kesirli kısmı olmak üzere, $$\int_{0}^{1} \left \{\dfrac{1}{x}\right \}^2-\left \{\dfrac{1}{x}\right \}dx=?$$
« Son Düzenleme: Ekim 10, 2019, 07:10:02 ös Gönderen: metonster »
Gerçek hikayeler aslında söylenmeyenlerdir.

Çevrimdışı Metin Can Aydemir

  • G.O Genel Moderator
  • Geo-Maniac
  • ********
  • İleti: 1139
  • Karma: +9/-0
Ynt: Kesirli Kısmın İntegrali
« Yanıtla #1 : Eylül 05, 2019, 02:03:48 öö »
Denklemi ayrı ayrı ele alalım, $\displaystyle I= \int_{0}^{1} \left \{\dfrac{1}{x}\right \}^2-\left \{\dfrac{1}{x}\right \} dx$, $\displaystyle I_1=\int_{0}^{1} \left \{\dfrac{1}{x}\right \} dx$ ve $\displaystyle I_2=\int_{0}^{1} \left \{\dfrac{1}{x}\right \}^2 dx$ olsun. $x=\dfrac{1}{t}$ dönüşümü yaparsak,$\lfloor x\rfloor$ tam kısım olmak üzere, $$I_1=\int_{0}^{1} \left \{\dfrac{1}{x}\right \} dx=\int_{1}^{\infty} \dfrac{\{t\}}{t^2} dt=\int_{1}^{\infty} \dfrac{t-\lfloor t\rfloor}{t^2} dt =\sum_{k=1}^{\infty}\int_{k}^{k+1} \dfrac{t-k}{t^2} dt=\sum_{k=1}^{\infty} \left (\ln \left (\dfrac{k+1}{k}\right )-\dfrac{1}{k+1}\right )$$

$$I_2=\int_{0}^{1} \left \{\dfrac{1}{x}\right \}^2 dx=\int_{1}^{\infty} \dfrac{\{t\}^2}{t^2} dt=\int_{1}^{\infty} \dfrac{(t-\lfloor t\rfloor)^2}{t^2} dt$$ $$\Rightarrow I_2=\sum_{k=1}^{\infty}\int_{k}^{k+1} \dfrac{t^2-2tk+k^2}{t^2} dt=\sum_{k=1}^{\infty} \left (2-2k\ln\left (\dfrac{k+1}{k}\right )-\dfrac{1}{k+1}\right)$$ $$I=I_2-I_1=\sum_{k=1}^{\infty} \left (2-(2k+1)\ln \left (\dfrac{k+1}{k}\right )\right )$$ $\displaystyle S_n=\sum_{k=1}^{n} \left (2-(2k+1)\ln \left (\dfrac{k+1}{k}\right )\right )$ olsun. $\displaystyle I=\lim_{n\to\infty} S_n$ olur. $$S_n=\sum_{k=1}^{n} \left (2-(2k+1)\ln \left (\dfrac{k+1}{k}\right )\right)=2n-\ln \left (\prod_{k=1}^{n}\dfrac{(k+1)^{2k+1}}{k^{2k+1}}\right )=2n-\ln \left (\dfrac{(n+1)^{2n+1}}{(n!)^2}\right )$$ $$I=\lim_{n\to\infty} \left (2n-\ln \left (\dfrac{(n+1)^{2n+1}}{(n!)^2}\right )\right)=\ln(2\pi)-2$$ olur. Bu limitin doğruluğunu wolframalpha'dan kontrol edebilirsiniz.
« Son Düzenleme: Ekim 10, 2019, 07:26:14 ös Gönderen: metonster »
Gerçek hikayeler aslında söylenmeyenlerdir.

Çevrimdışı Eray

  • G.O Genel Moderator
  • G.O Efsane Üye
  • ********
  • İleti: 414
  • Karma: +8/-0
Ynt: Kesirli Kısmın İntegrali
« Yanıtla #2 : Eylül 05, 2019, 07:45:54 öö »
$$I=\lim_{n\to\infty} (2n-\ln(\dfrac{(n+1)^{2n+1}}{(n!)^2}))=\ln(2\pi)-2$$ olur. Bu limitin doğruluğunu wolframalpha'dan kontrol edebilirsiniz.

Peki WolframAlpha'nın yanılıyor olmadığından nasıl emin olacağız? :P

Çevrimdışı AtakanCİCEK

  • G.O Demirbaş Üye
  • ******
  • İleti: 264
  • Karma: +4/-0
  • Manisa
Ynt: Kesirli Kısmın İntegrali
« Yanıtla #3 : Eylül 05, 2019, 01:03:51 ös »
Denklemi ayrı ayrı ele alalım, $I=\int_{0}^{1} \{\dfrac{1}{x}\}^2-\{\dfrac{1}{x}\} dx$, $I_1=\int_{0}^{1} \{\dfrac{1}{x}\} dx$ ve $I_2=\int_{0}^{1} \{\dfrac{1}{x}\}^2 dx$ olsun. $x=\dfrac{1}{t}$ dönüşümü yaparsak,$\lfloor x\rfloor$ tam kısım olmak üzere, $$I_1=\int_{0}^{1} \{\dfrac{1}{x}\} dx=\int_{1}^{\infty} \dfrac{\{t\}}{t^2} dt=\int_{1}^{\infty} \dfrac{t-\lfloor t\rfloor}{t^2} dt=\sum_{k=1}^{\infty}\int_{k}^{k+1} \dfrac{t-k}{t^2} dt=\sum_{k=1}^{\infty} (\ln(\dfrac{k+1}{k})-\dfrac{1}{k+1})$$

$$I_2=\int_{0}^{1} \{\dfrac{1}{x}\}^2 dx=\int_{1}^{\infty} \dfrac{\{t\}^2}{t^2} dt=\int_{1}^{\infty} \dfrac{(t-\lfloor t\rfloor)^2}{t^2} dt$$ $$\Rightarrow I_2=\sum_{k=1}^{\infty}\int_{k}^{k+1} \dfrac{t^2-2tk+k^2}{t^2} dt=\sum_{k=1}^{\infty} (2-2k\ln(\dfrac{k+1}{k})-\dfrac{1}{k+1})$$ $$I=I_2-I_1=\sum_{k=1}^{\infty} (2-(2k+1)\ln(\dfrac{k+1}{k}))$$ $S_n=\sum_{k=1}^{n} (2-(2k+1)\ln(\dfrac{k+1}{k}))$ olsun. $I=\lim_{n\to\infty} S_n$ olur. $$S_n=\sum_{k=1}^{n} (2-(2k+1)\ln(\dfrac{k+1}{k}))=2n-\ln(\prod_{k=1}^{n}\dfrac{(k+1)^{2k+1}}{k^{2k+1}})=2n-\ln(\dfrac{(n+1)^{2n+1}}{(n!)^2})$$ $$I=\lim_{n\to\infty} (2n-\ln(\dfrac{(n+1)^{2n+1}}{(n!)^2}))=\ln(2\pi)-2$$ olur. Bu limitin doğruluğunu wolframalpha'dan kontrol edebilirsiniz.

Sondaki limitte stirling formülünü yani  $n$ sonsuz iken $n!=\dfrac{n^n}{e^n}.\sqrt{2 \pi n}$ özdeşliğini kullanıp l hospitalleri ardı ardına uyguladım fakat dünyanın işlemi çıktı bence çok uğraşmayın derim. Dolayısıyla wolfram alphaya güvenebiliriz  :P
« Son Düzenleme: Eylül 05, 2019, 04:36:57 ös Gönderen: AtakanCİCEK »
Bir matematikçi sanmaz fakat bilir, inandırmaya çalışmaz çünkü ispat eder.
    Boğaziçi Üniversitesi - Matematik

Çevrimdışı Metin Can Aydemir

  • G.O Genel Moderator
  • Geo-Maniac
  • ********
  • İleti: 1139
  • Karma: +9/-0
Ynt: Kesirli Kısmın İntegrali
« Yanıtla #4 : Eylül 05, 2019, 02:40:05 ös »
Sizler için hemen göstereyim, Atakan aslında doğru yerden gitmiş ama uzun yoldan kullanmış galiba, Atakan'ın da verdiği Stirling formülünde her tarafın logaritması alınıp düzenlenirse, $$2\ln(n!)\approx \ln(2\pi)+(2n+1)\ln(n)-2n$$ elde edilir. Bunu limite yazarsak, $$I=\lim_{n\to\infty} \left ( 2n-\ln\left (\dfrac{(n+1)^{2n+1}}{(n!)^2}\right )\right )=\ln(2\pi)-\lim_{n\to\infty} \left (2n\ln \left (\dfrac{1}{n}+1\right )+\ln\left (\dfrac{1}{n}+1\right )\right )=\ln(2\pi)-2\lim_{n\to\infty} n\ln\left (\dfrac{1}{n}+1\right )$$ Şimdi $\displaystyle \lim_{n\to\infty} n\ln\left (\dfrac{1}{n}+1\right )$ limitinde $\dfrac{1}{n}=t$ dönüşümü yaparsak, $$\lim_{n\to\infty} n\ln\left (\dfrac{1}{n}+1\right )=\lim_{t\to 0^{+}} \dfrac{\ln(t+1)}{t}=\lim_{t\to 0^{+}} \sum_{k=1}^{\infty} \dfrac{(-1)^{k+1}\cdot t^{k-1}}{k}=\lim_{t\to0^{+}} t^0=1$$ Buradan da $I=\ln(2\pi)-2$ bulunur.
« Son Düzenleme: Ekim 10, 2019, 09:50:14 ös Gönderen: metonster »
Gerçek hikayeler aslında söylenmeyenlerdir.

Çevrimdışı Metin Can Aydemir

  • G.O Genel Moderator
  • Geo-Maniac
  • ********
  • İleti: 1139
  • Karma: +9/-0
Ynt: Kesirli Kısmın İntegrali
« Yanıtla #5 : Eylül 05, 2019, 05:01:26 ös »
Bu sorudan yola çıkarak $\displaystyle \int_{0}^{1} \left \{\dfrac{1}{x}\right \} dx$ ve $\displaystyle \int_{0}^{1} \left \{\dfrac{1}{x}\right \}^2 dx$ integrallerini de hesaplayabiliriz. Şimdi bununla uğraşalım. Sorunun ispatından da gösterdiğim gibi, $$I_1=\int_{0}^{1} \left \{\dfrac{1}{x}\right \} dx=\sum_{k=1}^{\infty} \left (\ln\left (\dfrac{k+1}{k}\right )-\dfrac{1}{k+1}\right )$$ Şimdi seri toplamı şeklinde yazalım. $$S_n=\sum_{k=1}^{n} \left (\ln\left (\dfrac{k+1}{k}\right)-\dfrac{1}{k+1}\right )$$ dersek, $\displaystyle H_n=\sum_{i=1}^{n} \dfrac{1}{i}$ olmak üzere, $$S_n=\ln\left (\prod_{k=1}^{n} \dfrac{k+1}{k}\right )-H_{n}+1=\ln(n+1)-H_n+1$$ $\gamma$, euler-mascheroni sabiti olmak üzere, tanımı gereği $$\gamma = \lim_{n\to\infty} (H_n-\ln(n))$$'dir. Şimdi $I_1$'i hesaplayalım. $$I_{1}=\lim_{n\to\infty} \left (\ln(n+1)-H_n+1\right )=1-\lim_{n\to\infty} \left ( H_{n+1}-\ln(n+1)-\dfrac{1}{n+1}\right )=1-\gamma$$ olur. Soruda da ispatladığımız gibi $I_2-I_1=\ln(2\pi)-2$'dir. Buradan da, $$I_2=\int_{0}^{1} \left \{\dfrac{1}{x}\right \}^2 dx=\ln(2\pi)-\gamma-1$$ bulunur.
« Son Düzenleme: Ekim 10, 2019, 09:55:14 ös Gönderen: metonster »
Gerçek hikayeler aslında söylenmeyenlerdir.

 


Sitemap 1 2 3 4 5 6 7 8 9 10 11 12 13 14 15 16 17 18 19 20 21 22 23 24 25 26 27 28 29 30 31 32 33 34 35 36 37 
SimplePortal 2.3.3 © 2008-2010, SimplePortal